How Do You Solve a Logistic Growth Model Problem in Population Dynamics?

  • Thread starter Thread starter BraedenP
  • Start date Start date
  • Tags Tags
    Integration Model
Click For Summary
The discussion focuses on solving a logistic growth model for population dynamics, specifically using the equation dy/dt = ky(1 - y/C). Given the population data for the years 1987 and 1999, the user successfully rearranged and integrated the equation to find a value for the constant k. However, they encounter difficulties in calculating the future populations for 2050 and 2100 due to the complexity of logarithmic expressions. The user seeks guidance on how to proceed with these calculations, particularly for the scenario where the carrying capacity changes. The thread emphasizes the challenges of applying mathematical transformations in logistic growth models.
BraedenP
Messages
94
Reaction score
0

Homework Statement



The following is a population logistics model where t is the year, C is the carrying capacity, k is some unknown constant, and y(t) is the population at time t.

I am given \frac{dy}{dt}=ky\left ( 1-\frac{y}{C} \right)
and that y(t) is the population size for the year t.

I am also given that when t=1987, y=5 AND when t=1999, y=6. C is always 20.

I am supposed to figure out (a) the population in 2050 where C=20, and (b) the population in 2100 where C=10.

Homework Equations



The Attempt at a Solution



I rearranged the equation to form: \frac{1}{y}+\frac{1}{C-Y} dy = k dt
and then integrated to get: ln(y)-ln(C-Y)=kt+N, (where N is the constant of integration to avoid confusion with C)

This is where things start going wrong. I substituted the "1987" and "5" values into the equation and solved for N, getting: N=ln(\frac{1}{3})-1987k and then substituted this equation back into the equation using the other two values, "1999" and "6":

ln(\frac{3}{7})=1999k + ln(\frac{1}{3})-1987k

I then solved for k, getting: k=\frac{ln(\frac{9}{7})}{12}

All of these logarithms prevent me from calculating a nice clean answer for (a), and I have no clue where to start with regard to (b). Can anyone point me in the right direction?

Thanks!
 
Physics news on Phys.org
at this point how about
<br /> ln(y)-ln(C-y)<br /> =ln(\frac{y}{C-y}) = kt + N<br />

<br /> \implies \frac{y}{C-y} = e^{kt + N}<br />

<br /> \implies y(1+e^{kt + N}) = Ce^{kt + N}<br />

<br /> \implies y <br /> = \frac{Ce^{kt + N}}{1+e^{kt + N}} <br /> = \frac{C}{e^{-(kt+N)}+1}<br /> = \frac{C}{Be^{-kt}+1}<br />

where B is a constant
 
Last edited:
Question: A clock's minute hand has length 4 and its hour hand has length 3. What is the distance between the tips at the moment when it is increasing most rapidly?(Putnam Exam Question) Answer: Making assumption that both the hands moves at constant angular velocities, the answer is ## \sqrt{7} .## But don't you think this assumption is somewhat doubtful and wrong?

Similar threads

  • · Replies 2 ·
Replies
2
Views
1K
  • · Replies 3 ·
Replies
3
Views
1K
Replies
4
Views
2K
Replies
5
Views
2K
  • · Replies 1 ·
Replies
1
Views
1K
Replies
3
Views
2K
Replies
12
Views
2K
  • · Replies 3 ·
Replies
3
Views
2K
  • · Replies 12 ·
Replies
12
Views
3K
  • · Replies 2 ·
Replies
2
Views
3K